Anda di halaman 1dari 44

SECTION 1

Quantitative

QUANTITATIVE

GMAT DIAGNOSTIC SECTION 1: QUANTITATIVE


Total Questions: 37 Time: 75 Minutes

Directions for Section One: The following questions are either Problem Solving or Data Sufficiency. Problem Solving questions are those with five listed answer choices. In each of these, select the best answer choice, and mark the corresponding oval on your exam grid. Note that figures are drawn as accurately as possible except when it is stated that a figure is not drawn to scale. All figures lie in a plane unless otherwise indicated. Data Sufficiency questions are those which have a question followed by two statements containing certain data. Your task is to determine whether the data provided by the statements are sufficient to answer the question. The answer choices for all of these questions are always the same. They are the following. (A) Statement (1) BY ITSELF is sufficient to answer the question, but statement (2) by itself is not. (B) Statement (2) BY ITSELF is sufficient to answer the question, but statement (1) by itself is not. (C) Statements (1) and (2) TAKEN TOGETHER are sufficient to answer the question, even though NEITHER statement BY ITSELF is sufficient. (D) EITHER statement BY ITSELF is sufficient to answer the question. (E) Statements (1) and (2) TAKEN TOGETHER are NOT sufficient to answer the question, requiring more data pertaining to the problem. Based upon the statements data, your knowledge of mathematics, and your familiarity with everyday facts (such as the number of minutes in an hour), select the best answer choice, and mark in on your answer grid. A figure accompanying a Data Sufficiency question will conform to the information in the stem, but will not include data from the statements. Figures are not necessarily drawn to scale. Note that you may assume that the positions of points, angles, regions, etc., exist in the order shown and that angle measures are greater than zero. All figures lie in a plane unless otherwise indicated.

GMAT DIAGNOSTIC 1.
1 2

1 3 1 4 1 20 1 4 9 20 29 60

1 4

1 5

1 6

G G 1 G 1 G 1 G 1

y w x

2. A gambler bought $3,000 worth of chips at a casino in denominations of $20 and $100. That evening, the gambler lost 16 chips, and then cashed in the remainder. If the number of $20 chips lost was 2 more or 2 less than the number of $100 chips lost, what is the largest amount of money that the gambler could have received back? G $2,040 G $2,120 G $1,960 G $1,920 G $1,400 3. Was Madelines job interview on a Tuesday? (1) Exactly 50 hours before the interview, it was Sunday. (2) The interview was between 2:00 p.m. and 10:00 p.m.

The circular base of a planter sits on a level lawn, and just touches two straight garden walls at points w and y. The walls come together at point x, which is 15 inches from the center of the planter. What is the area of the base of the planter? (1) Both points Y and W are 9 inches from the center of the planter. (2) The garden wall touches the planter at 12 point inches from point X.

5. Paul has s fewer dollars than Jane has, and together they have a total of t dollars. Which of the following represents the number of dollars that Paul has? G G
ts 2 t 2

s
s 2

G t

G 2s t G s 2t 6. A mathematics department has 5 members. The department must send a team of 2 members to a conference. If there is one pair of members who refuse to attend together, how many possible pairs of department members can be chosen? G 14 G G G G 9 7 5 3

QUANTITATIVE 7. A realtor sells homes for 20% above the initial acquisition cost. If a house does not sell within six months, the realtor will then mark the price down to only 5% above acquisition cost. A house has just entered the market and the realtor has priced it at $360,000. What will the reduced price be in dollars if the house does not sell within six months? G 302,400 G 306,000 G 315,000 G 318,000 G 342,000 8. 5(60) + 4(121) G 103 G 22 G 28 G 31 G 103 + 21 9. If s + t = 10, what is the value of q r ? (1) r = s (2) tq sr + sq tr = 20 10. Is a = c? (1) a + b = b c (2) c a 11. Is 3x < 300? (1) 3x 1 < 250 (2) 3
x+1

13. If x *y = 2(x 2)(y 3), which of the following represents possible values for (x,y ) when x *y = 1,350? G (3, 5) G (5,3) G (10,5) G (13,5) G (13,50) 14. The variables a, b, and c are distinct factors of 24. Which of the following must be true? G a + b + c is even. G a + b + c < 40 G G abc is even. G abc is a multiple of 24. 15. ( 8 1)(1 8 ) = G 2 29 G 2 28 G 4 29 G 2 2 G 4 2 16. If Aishas age is exactly a third of Sukis, what is Aishas age? (1) In 15 years, Aisha will be as old as Suki. (2) 5 years ago, Suki was 4 times older than Aisha.
a 5 = , and 17. The integers a, b, and c are positive, b 2 a 7 = . What is the smallest possible value of c 5 2a + b? ab c

is an integer.

= 3 + 486

12. If 100 cars drive down 3rd Avenue every hour, three times as many cars use 2nd Avenue as 1 of the cars on both use 3rd Avenue, and 4 avenues are taxis, how many taxis drive down 2nd Avenue every hour? G G G G 25 50 60 75

G G G G

63 70 84 95

G 105

G 200

GMAT DIAGNOSTIC 18. It takes John 3 hours to assemble a model plane. Working together, Mike and John take 2 hours to assemble the model plane. How long would it take Mike working alone to assemble the plane? G 2 G 3 G 4 G 5 G 6 19. The length of rectangular box is increased by 20 percent while its width is decreased by 20 percent. The volume of the altered rectangular box is what percent of the volume of the original rectangular box? G G 60% 96% 22. If
5 s

= 3 and
7 45 1 5 17 45 9 5 17 5

t 5

= 4, then

4+s t5

G G G G G

23. Each of a, b, and c is a positive integer, 3b < a, and a + b = c. What is the value of b ? (1) c = 8 (2) a = 7

G 100% G 140% G 144% 20. What is the value of x ? (1) x 2 + 4x = 8x 35 (2) x 2 + 3x = 7x 25 21. A bag has only r red marbles and b blue marbles. If five red marbles and one blue marble are added to the bag, and if one marble is then selected at random from the bag, then the probability that the marble selected will be blue is represented by G G G G G
b r b b+r b+1 r+5 b+1 r+b+5 b+1 r+b+6

(0, 2) (0, 1) ,1) (1 2 (1, 0)

(0, 0)

24. In the rectangular coordinate system above, the shaded region is bounded by straight lines. Which of the following is NOT an equation of one of the boundary lines? G y=0 G y=1 G x=0 G y = 2x + 2 G y 2x = 1 25. Is r > 4 ? (1) r 2 3r 10 = 0 (2) r (r 5)(r 6) = 0

QUANTITATIVE 26. A commuter train stops at stations M, N, O, and P it that order. What is the distance along the railroad tracks between stations N and O ? (1) The distance along the railroad tracks between stations M and O is 15 miles. (2) The distance along the railroad tracks between stations N and P is 12 miles. 27. If a and b are prime numbers, which of the following must also be a prime number? G a+b G ab
1 ) G (a 2b )(

Initial Cost Car Service X Car Service Y $14 $10

Cost per Mile $1.20 $1.28

30. The table above shows the initial cost and cost per mile for the distance traveled when using car service X and car service Y. What distance, in miles, will cost the same with the two car services? G 42 G 50 G 68 G 75 G 84 31. What is the value of the two-digit integer a ? (1) The sum of the two digits of a is 9.

ab 2 2 ) (a b )( 1 ab

G a 2b 2 ab

12

(2) a is a multiple of 3.
x+y = 4, what does x equal, in 32. If x y and xy terms of y? 5 y G

14

b 28. In the figure above, the sum of the numbers in the vertical column equals the sum of the numbers in the horizontal row. What is the value of a? G 2

G y G
3 y 5

G y G
5 y 3 (53) y? (52)
x

G 12 G 14 G 26 G 40 29. In triangle ABC, is AB < 12 ? (1) The length of AC is 7. (2) The length of BC is 10.

33. Question: If x 0, what is the value of (1) x = y (2) x is a positive integer.

34. In a certain community orchestra, 40 percent of the members are string players. If 28 percent of the members are female string players, how many of the members are female string players? (1) Exactly 120 of the members are string players. (2) Exactly 35 percent of the players who are not string players are male.

GMAT DIAGNOSTIC

35. Due to a manufacturing defect, each copy of a 300-page book is missing exactly one page, though each copy is not missing the same page. If the text of the book is only 288 pages (the other 12 are blank), what is the probability that any single copy is missing a page of text? G G G G G
1 30 1 25 11 12 14 15 24 25

36. The sum of 18 hours, 20 minutes and 6 hours, 40 minutes is what fraction of a week? G G G G
1 7 25 168 363 1000 247 500

G 1 37. If m is a positive integer, is m + 4 even? (1) 2m + 1 is odd. (2) 2m + 4 is a multiple of 4. END OF SECTION ONE

QUANTITATIVE

ANSWERS ON THE NEXT PAGE

GMAT DIAGNOSTIC

Section 1: Quantitative Answer Key


1. 2. 3. 4. 5. 6. 7. 8. 9. 10. 11. 12. 13. 14. 15. 16. 17. 18. 19. 20. 21. 22. 23. 24. 25. 26. 27. 28. 29. 30. 31. 32. 33. 34. 35. 36. 37. D B C C A B C C B A B D B D C D C E B B E C A E C E C B E B E E E A E B B

10

SECTION 2

Verbal

VERBAL

GMAT DIAGNOSTIC SECTION 2: VERBAL


Total Questions: 41 Time: 75 Minutes
Directions for Section Two: This section consists of Sentence Correction questions, Critical Reasoning questions, and Reading Comprehension questions. Sentence Correction questions consist of sentences that are either partly or entirely underlined. Below each sentence are five versions of the underlined portion of the sentence. The first of these, choice (A), duplicates the original version. The four other versions revise the underlined portion of the sentence. Read the sentence and the five choices carefully, and select the best version. If the original version seems better than the revised versions, select the first choice, (A). If not, choose one of the revised versions. Choose answers according to the norms of standard written English for grammar, word choice, and sentence construction. Your selected answer should express the intended meaning of the original sentence as clearly and precisely as possible, while avoiding ambiguity, awkwardness, or unnecessarily wordy constructions. Critical Reasoning questions consist of a set of statements, followed by a question. Analyze the statements, then select the answer choice that is the most appropriate response to the question. No specialized knowledge of any particular field is required for answering these questions. Reading Comprehension questions refer to a specified passage. Your choice is to be based on what is stated or implied in the passage, and not on your own knowledge. You may refer to the passage while answering the questions.

1. Developed in 1919 in the work of the German scholar Karl Schmidt, the method of literaryhistorical analysis known as Form Criticism continued to be influential well into the 1980s. G Developed in 1919 in the work of the German scholar Karl Schmidt G The work of the German scholar Karl Schmidt was developed in 1919 G Developed in 1919, the German scholar Karl Schmidts work G Develops in 1919 in the work of the German scholar Karl Schmidt G In the work of the German scholar Karl Schmidt

2. Methotrexate, which was originally developed for a single purpose, has proven its effectiveness in several applications, including treating cancer, relieving arthritis, and it is able to dull pain. G including treating cancer, relieving arthritis, and it is able to dull G including treating cancer, relieving arthritis, and dulling G including the treatment of cancer, relieving arthritis, and dulling G which includes treating cancer, relieving arthritis, and it is able to dull G which includes treating cancer, relieving arthritis, and dulling

13

GMAT DIAGNOSTIC

3. Deep vein thrombosis is a medical condition characterized by painful and often life-threatening clogs in the veins. Researchers who examined 180 people with recurrent deep vein thrombosis and 200 without this ailment found that those with high levels of homcysteine, an amino acid, were three times more likely to have deep vein thrombosis than were those with average homocysteine levels. Based on these results, physicians recommend that people with high homocysteine levels increase their consumption of folate, a nutrient that reduces clotting by lowering levels of homocysteine. Which of the following, if true, most indicates that the physicians recommendation, if acted upon, would have the desired result? G Those in the study with deep vein thrombosis also had irregular levels of s everal other amino acids. G People who consume high amounts of folaterich foods such as orange juice, spinach, and broccoli have been shown to have below-average homocysteine levels. G People whose daily consumption of folate is less than that recommended by physicians are more prone to heart attacks and strokes than are people who consume the recommended daily allowance of this nutrient. G If the clogs caused by deep vein thrombosis are left untreated, they may eventually lodge in the lungs and cause sudden death. G In a series of large, well-designed medical studies, the consumption of high levels of folate has not been associated with any serious health risks.

Questions 4 through 7 are based on the following passage: The relationships between age, cognitive functioning and depression seem to be intricate and dauntingly cryptic. One of the most serious obstacles to accurate assessments of cognitive performance of the elderly lies in the overlap between the symptoms of depression and dementia, a mental disorder characterized by the loss of cognitive function, which has made differential diagnosis of dementia and depression an ongoing (and formidable) challenge. Many of the subjective and vegetative symptoms of depression, such as apathy and sleep disturbances, also occur in individuals in the early stages of dementia. Reciprocally, the symptoms of depression include cognitive difficulties with concentration, decisionmaking, and memory. Because depression can be effectively treated in a variety of ways, accurate differential diagnosis is essential. Antidepressants will generally improve the cognitive dysfunction associated with depression but will have minimal effect on organic degeneration. Accurate diagnosis is most elusive in the early stages of depression and dementia. The traditional methods of delineating between the two have relied on symptoms, psychiatric history, family history of depression, rate of onset, and response to treatment. Decreased appetite is a common symptom of depression, but many Alzheimers patients actually report increased eating in the early stages of the disorder. Olfactory problems are frequent in neurodegenerative disorders, but depressed individuals almost never exhibit a decreased sense of smell. Interestingly, the time between onset of symptoms and seeking of treatment may also provide a clue: research has shown that the median duration of cognitive problems before treatment is approximately five years for demented individuals but a scant two months for depressed elderly. This may be reflective of the protracted degeneration of dementia.

(5)

(10)

(15)

(20)

(25)

(30)

(35)

(40)

A considerable amount of research has been devoted to differences in how depressed and demented elderly respond to cognitive testing. In (45) memory tasks, depressed patients have exhibited a tendency towards a conservative response bias and reluctance to guess. Conversely, dementia patients are far more likely to venture an answer even when they are unsure of its accuracy. (50) Typically, elderly depressed subjects perform at a

14

VERBAL level between that of normal controls and dementia subjects on cognitive test batteries. This suggests that both depression and dementia exacerbate the cognitive declines associated with normal aging (55) and that the difference is one of degree. 7. According to the passage, all of the following are ways to distinguish between depression and dementia in the elderly EXCEPT: G sensory functioning G less than normal performance on cognitive tests 4. The authors primary purpose is to G discuss problems that arise from a failure to distinguish between depression and dementia G explicate the difficulties in differentiating between dementia and depression in the elderly G contrast the efficacy of cognitive testing with other diagnostic methods G lobby for funding for research into more effective methods of cognitive assessments G describe the cognitive performance of normal, depressed, and demented elderly 5. Which of the following are cited by the passage as reasons why it is difficult to differentiate between dementia and depression? I. II. III. Similar symptoms are manifest in the early stages of both disorders. People with dementia have an increased vulnerability to depression. Dementia and depression have the same, underlying organic cause. G willingness to guess during cognitive testing G duration of tolerance for symptoms G response to antidepressant medication 8. Paleontologists have difficulty accurately reconstructing early Cambrian ecosystems, since most animal species in the early Cambrian era lacked hardened body parts. When an animal dies, its soft parts decayed very rapidly, leaving only the harder parts, typically bones, teeth, or shell. If the animal is buried quickly enough after death, these parts can become fossilized. If an animal lacked hardened body parts, its remains only become fossilized under exceptional circumstances. If the above is true, which of the following can be properly concluded from these statements? G When reconstructing an early Cambrian ecosystem, paleontologists must take into account that some animals that lived in that ecosystem may have left no trace in the fossil record. G There were more living species with hardened body parts than with only soft body parts in the early Cambrian era. G There are no traces in the fossil record of early Cambrian animals that lacked bones, teeth, or shell. G In an early Cambrian ecosystem containing hard-shelled trilobites and soft bodied priapullid worms, only the remains of the trilobites would be preserved in the fossil record. G It is impossible to reconstruct early Cambrian ecosystems accurately.

G I only G II only G III only G I and II G II and III 6. Why does the author discuss olfactory impairment in the second paragraph? G To bolster his assertion that the symptoms of depression and dementia overlap G To demonstrate that dementia has an underlying, organic genesis while depression does not G To provide a concrete example of a distinguishing symptom of dementia G To illustrate the breadth of symptoms associated with neurodegenerative disorders G To cite an example of sensory impairment common in the elderly

15

GMAT DIAGNOSTIC 9. During this fiscal year, Company X has suffered a marked decrease in productivity. Company managers attribute this to budget decisions that allocated more money to pay freelancers than to pay part-time employees, which resulted in the elimination of numerous part-time positions. The freelance budget has gone mostly to new hires, who have less training, on average, than parttimers and it would take a substantial time to train the freelancers. Assuming that the managers reasoning is sound, which of the following, if feasible, is most likely to mitigate the problem raised by the managers? G Increasing the work hours of the remaining part-time employees. G Providing performance bonuses for managers whose departments demonstrate increased productivity. G Reallocating part of the freelance budget to pay for efficiency consultants. G Rehiring former part-time workers as freelancers. G Eliminating departments with particularly low productivity. 10. The benefits of having many trees on ones property range from insulation and energy conservation in summer and winter, cleaning the air, and increases property value. G cleaning the air, and increases G to cleaning the air, and increasing G cleaner air, and increases in G to cleaner air and increasing G to cleaner air and increased 11. Unusually-colored food items like green ketchup, pink margarine, and blue popcorn are aimed at consumers ages 517, but may soon influence adult foods as well. G are aimed at consumers G are aimed to consumers G are aimed to consumers G is aimed at consumers G is aimed to consumers 12. Johns old high school friend, who was once lively, energetic, and dynamic, is now idle, lazy, and is apathetic. G now idle, lazy, and is apathetic G now idle, lazy, and is now apathetic G now idle, lazy, and apathetic G idle now, along with being lazy and apathetic G idle, lazy, and apathetic now 13. An anesthetic that is highly effective and has no negative side effects can be produced only from the webs of a spider species that is endangered in the wild. It takes 1,000 webs to make 1 gram of the drug. It follows, therefore, that continued production of the drug must inevitably lead to the species extinction. Which of the following, if true, most seriously weakens the argument above? G The anesthetic made from the spiders webs is widely distributed. G The anesthetic made from the spiders webs is expensive to manufacture. G Parts from the spiders are used to produce other medications. G Each night the spiders spin new webs and abandon the old webs. G The spiders live in very inaccessible places.

16

VERBAL Questions 14 through 17 are based on the following passage: In journalism, bias is a word with many meanings. It suggests a single explanation conscious, even willful preference for a selective portrayal of a situationfor a range of instances in (5) which the message does not reflect the reality. But few objective observers of, for instance, the reporting of campaign finance would argue that conventional biases are operating there. Journalists in general are not singling out Democrats or (10) Republicans, liberals or conservatives, for praise or blame. Rather one has to look to more intrinsic and ingrained factorsto the structural biases of American newspapers and the political assumptions of their reporters, editors, and (15) headline-writersto explain bias in the news. Structural biases are rooted in the very nature of journalismin its professional norms, in marketplace imperatives, in the demands of communicating information to an unsophisticated (20) audience. Stories need identifiable actors, understandable activity, and elements of conflict, threat or menace. They cannot be long, and must avoid complexitymust focus on controversy, personalities and negative statistics rather than (25) on concepts. These define the good story. As for political assumptions, all observers bring a cognitive map to American politicsa critical posture toward politics, parties, and politicians. For some it is as simple as all politicians are (30) crooks; for others it involves understanding the distribution of power and influence in America. The medias particular understanding of the ways of influence and decision-making in government colors the way they describe political It also defines their responsibility in (35) reality. reporting that reality; contemporary reporters are in many ways the grandchildren of the Progressive muckrakers. Few aspects of American politics reinforce this Progressive world-view as effectively (40) as the American way of campaign finance. Its cash is an easy measure of influence, and its PACs are perfect embodiments of vested, selfish interests. In assuming that public officials defer to contributors more easily than they do to their party, (45) their own values, or their voting constituency, one has the perfect dramatic scenario for the triumph of wealthy special interests over the will of majorities and the public interest. Structural bias and political assumption, (50) finally, meet in an analytical conundrum. Structural biases dictate that newspapers print stories that will be read. But does the press publish a story because readers have been previously conditioned to accept and believe such (55) accounts, or does it publish the story because of its conviction that it represents political truth?

14. Which of the following best states the main idea of the passage? G Business decisions necessary in journalism lead to inherent biases. G The American public demands bad journalism. G Two factors influence journalists reporting on campaign financing. G Systematic bias and political assumption cause an analytical conundrum. G Only non-profit journalism can avoid journalistic biases. 15. The author suggests that structural biases in American journalism result primarily from: G problems intrinsic to the publishing and marketing of newspapers. G assumptions by journalists about the integrity of public officials. G reporters cynicism about the publics level of intelligence. G growing competition among newspapers for a shrinking audience. G the liberal biases generally found in publishing and broadcast media. 16. According to the passage, which of the following would demonstrate structural biases inherent in journalists work? I. II. III. An article that adheres loyally to Progressivist dictates An article that successfully masks its biased opinions An article that is entertaining and easily understood

G I only G I and II only G II only G II and III only G III only

17

GMAT DIAGNOSTIC 17. The author suggests in the passage that the American system of campaign finance G is unjust and should be reformed. G has exclusively served the interests of the wealthy. G is an easy target for journalists. G has been unfairly singled out for criticism by politicians. G can never overcome its inherent biases. 18. Which of the following best describes the analytical conundrum referred to in the fourth paragraph? G Newspapers cynically promote Progressive ideas in which they do not believe. G It is difficult to distinguish the roles of structural biases and political assumptions in publishing decisions. G Structural biases and political assumptions exert conflicting pressures on newspaper publishers. G Readers preferences for dramatic news accounts reflecting Progressive ideas determine what is published. G Structural biases are rooted in the journalisms professional norms. 19. The author would most probably agree that the medias evaluation of the effects of PACs on public officials is G fair minded G sophisticated G uninformed G simplistic G mistaken 20. Noting the success of DVDs in the marketplace, the newly opened video store Movie Tapes, which sells specialty videocassettes, began an ambitious publicity campaign to promote the sale of their tapes over more technologically advanced DVDs. After six months in business, Movie Tapes found that its sales had doubled in fact, it had sold twice as many copies of movies on tape as all the other stores in town had sold on DVD of the same movies. Clearly, the publicity campaign was responsible for Movie Tapes video cassettes selling better than DVDs. Which of the following, if true, casts the most serious doubt on the conclusion drawn above? G DVDs include special features that enhance the viewing experience. G The local school system uses only VCRs in the classroom. G Only a very small number of the movies the store sells are available on DVD. G A month before the campaign began, the public library started buying videocassettes to lend. G Neither the price of DVDs nor that of videocassettes has risen in two years. 21. In the arena of litigation alternatives, one advantage of collaborative law over mediation is that the pre-trial outcome is decided by lawyers and litigants, working together in full participation throughout the process, instead of a mediator. G instead of G in contrast to G rather than by G and not decided by G compared to being done by

18

VERBAL 22. In Nation X, the average price of a new automobile is 15,000 to 20,000 US dollars, and the average automobile loses approximately ten percent of its market resale value each year. Therefore, if a two-year-old automobile is resold for only 10,000 US dollars, its loss in market resale value has been greater than average for Nation X. Which of the following indicates a flaw in the reasoning above? G Market resale value is only one measure of an automobiles value. G Some two-year-old automobiles in Nation X have market resale values in excess of 20,000 US dollars. G It is possible for a new automobile in Nation X to cost 30,000 US dollars. G A loss in market resale value beyond what is ordinarily expected does not necessarily indicate poor design. G Not all new automobiles are sold for a price within the average range. 23. Zeta Manufacturing, a producer of low-cost chairs and tables, plans to start production on a more expensive line of furniture. The companys research shows that consumers are currently reluctant to purchase products made by Zeta Manufacturing because of concerns about the products quality. The company is certain that selling a more expensive line will alleviate these concerns. Which of the following is an assumption that lies behind Zeta Manufacturings decision to produce a more expensive line of furniture? G Consumers believe that items that are equal in quality are equal in cost. G Consumers make a connection between the cost of a product and its quality. G Consumers are less attracted to low cost than they are to high quality. G Companies increase their profitability when they sell more expensive products. G People expect to purchase furniture infrequently because furniture is durable. 24. For the first time, an earth-like planet has been discovered. This planet is orbiting another star. Since we can now say with confidence that at least some of the 100 billion other stars like our sun may have planets like earth, the greatest chance for extraterrestrial life to exist is outside our solar system. The argument above assumes which of the following? G Extraterrestrial organisms are likely to appear in forms similar to terrestrial ones. G Life cannot exist on the other planets of our solar system. G Life will always occur wherever earth-like planets exist. G Life is most likely to arise under earth-like conditions. G Life from other kinds of planets is too difficult to detect. 25. The Earths inner core, a ball of solid iron, may be rotated slower than by the outer Earth. G may be rotated slower than by the outer Earth G the outer Earth may rotate more slowly than it G it may rotate slower than the outer Earth G may rotate more slowly than the outer Earth G may be more slow at rotating than the outer Earth

19

GMAT DIAGNOSTIC 26. A program instituted in a particular state allows residents aged 18 to 39 to prepay at current prices for any future healthcare needs that arise after age 65. The program then absorbs the additional costs of any such treatment if the treatment is sought at a state-run healthcare facility. The states residents should enroll in the program as a means of defraying the cost of their future medical needs. Which of the following, if true, is the most appropriate reason for a resident of the state not to participate in the program? G Many residents are unsure about where in the state they will retire, and therefore which state-run healthcare facility they will eventually use. G For more than eighty-nine percent of state residents, the amount of money earned by putting the prepayment funds in an interestbearing account today will be greater than the total cost of future healthcare at a staterun facility. G The average cost of healthcare at the states facilities is expected to rise faster than the average income of state residents. G Some of the states healthcare facilities are contemplating significant cost increases next year. G The prepayment plan would not cover the cost of elective procedures, such as cosmetic surgery, even if the procedures are performed at state-run healthcare facilities. 27. The limousine driver has waited for almost an hour before deciding his client was not going to show up. G The limousine driver has waited for almost an hour before deciding G The limousine driver waited for almost an hour before deciding that G The limousine driver has waited for almost an hour before he decided that G The limousine driver was waiting for almost an hour before deciding G The limousine driver waited for almost an hour before deciding 28. Pandas in captivity, two who live in the National Zoo in Washington D.C., are being actively bred in hopes of developing a sustainable population to reintroduce into the wild. G two of who G and two that G including two of which G two of those that G two of which

20

VERBAL 29. Additional to being source of Vitamin C and Vitamin A, a tomato contain a carotenoid called lycopene that is thought to help protect against diseases such as cancer and heart disease. G Additional to being source of Vitamin C and Vitamin A, a tomato contain G In addition to being a source of Vitamin C and Vitamin A, tomatoes contain G As well as Vitamin C and Vitamin A, a tomato contains G Tomatoes not only are a source of Vitamin C and Vitamin A, but also contain G A tomato not only is a source of Vitamin C and Vitamin A, it also contains 30. In 1997, the evening adult education program at Bresgin University experienced a large decrease in enrollment by hotel and restaurant professionalsformerly a large segment of the programs students. The administration believes the drop is probably due to an increase in tuition for those courses that receive degree credit, since the program had previously charged the same tuition for credit and non-credit courses. Which of the following, if feasible, offers the best prospects for alleviating the decrease in enrollments for the evening adult education program, if the administration is correct? G Establish job-placement programs for students in courses relevant to hotel and restaurant professionals. G Conduct a publicity campaign to educate hotel and restaurant professionals on the benefits of adult education. G Offer credit courses in subjects relevant to hotel and restaurant professionals at the same price charged for non-credit courses. G Eliminate the disparity in the cost between credit and non-credit courses by increasing the price of the non-credit courses. G Establish a degree program made up entirely of credit courses from the evening adult education program.

21

GMAT DIAGNOSTIC Questions 31 through 34 are based on the following passage: Demand for computer chips skyrocketed with the demand for PCs (personal computers) in the first half of the 1990s, making these extremely lucrative years for the semiconductor (5) industry. With 1995 sales topping $150 billion, and with the release of powerful new software which required most PC users to upgrade their computers, sales were expected to continue to rise. However, late in 1995, the price of the most (10) common memory chip dropped by over a third, and the book-to-bill ratio for semiconductor manufacturers fell to around 0.9for every $100 worth of chips produced, only $90 worth of new orders were received. Ironically, it seems that worlds newest industry was susceptible to hog cycles, a phenomenon long recognized in its most ancient industryfarming. Producers tend to reduce their output when the demand for and price of the (20) commodity they produce falls. However, in agriculture, supplies cannot adjust quickly to demand, since growing a crop or fattening an animal takes many months. Farmers are forced to fix next years output on current prices, which can lead to (25) volatility in commodity prices. They cannot cut production even when demand drops, producing a glut and even lower prices. Conversely, when demand rises, there is often no means of quickly increasing production, leading to shortages and (30) high prices. Similarly, the long lead times needed to build new semiconductor fabrication plants means that the supply of chips does not adjust smoothly to price changes. In late 1995 consumers largely chose not to upgrade their computers and (35) abstained from using the newly released software; the anticipated rise in demand never occurred. The previous years large investment in plants yielded extra supplies of chips, and prices plunged.
(15)

32. Which of the following best describes the structure of the passage? G An situation is outlined and then a theory is posited to explain that event. G An event in one industry is described and discussed in the context of problems in similar industries. G A problem is outlined and a mechanism for controlling this problem is posited. G An event in one industry is explained by discussing analogous events in a different industry. G Related problems in two different industries are compared and contrasted. 33. It is most likely that the author states that for every $100 worth of chips produced, only $90 worth of new orders were received in order to G explain what is meant by the term book-tobill ratio G give a concrete example of the fall in chip prices G illustrate that the vagaries of chip prices were not anticipated by the semi-conductor industry G provide an example that would illustrate the action of the hog cycle G demonstrate that the anticipated rise in demand for chips did not occur 34. It can be inferred from the information in the passage that a sharp and unexpected increase in the demand for memory chips would result in G a rapid increase in the number of such chips produced, and a subsequent glut in the supply of such chips G reduced construction of new semiconductor fabrication plants since higher prices result in higher profits without increased investment G a scarcity of such chips and a subsequent rise in their sale price G a substantial decrease in the book-to-bill ratio for the semi-conductor industry G a short term upward adjustment in the price of chips, followed by a slow decrease in price as new fabrication plants come on-line

31. The primary purpose of this passage is to G argue for the development of more effective means of detecting long-term fluctuations in the price of semi-conductors G contrast the problems in high tech industries with problems in low-tech industries G show how a mechanism that leads to swings in the price of farm produce also leads to swings in the price of computer chips G prove that wild swings in the prices of semiconductors and farm produce are inevitable G show that primary and high-technology industries share many of the same problems

22

VERBAL 35. Having provided for lifes basic necessities and having seen their children off into the world, the Baby Boomers are now more affluent, with more cash left over for discretionary spending. G Having provided for lifes basic necessities and having seen their children off into the world G Having provided for lifes basic necessities and seeing their children off into the world G Having lifes basic necessities provided for and their children being seen off into the world G Although they have provided for lifes necessities and seen their children off into the world G Despite having provided for lifes basic necessities and having seen their children off into the world 36. Although it consisted of only three films, James Dean had a career that generated an enduring legend of him as a moody, unpredictable rebel. G James Dean had a career that G James Deans career had that G the career that was of James Dean G James Deans career G it was the career of James Dean that 37. Four patients taking the sleeping pill SleepSoon, manufactured by DrugRite, died during a sixmonth period, yet the government health agency of Country X decided not to remove SleepSoon from the market. Many industry observers were surprised, since the death of so many patients taking a particular drug will often lead to the drugs being removed from the market. Which of the following, if true, provides the best indication that the government health agencys decision was logically well supported? G In the previous two-year period, twelve patients taking SleepSoon died, while only two patients taking other sleeping pills died during the same time period. G The deaths of the patients taking SleepSoon were found to be unrelated to the sleeping pill SleepSoon. G The government health agency of Country X issued new guidelines for drug companies to help standardize the safety criteria that a drug must meet prior to being placed on the market. G Consumer groups lobbied for legislation requiring drugs to meet stricter safety standards before being placed on the market. G Many DrugRite employees left the company for jobs at other drug manufacturers. 38. Malteses, which originated in south central Europe, have long been a prized dog when ancient Egyptians are thought to have worshiped them and Queen Elizabeth I kept them as pets. G Malteses, which originated in south central Europe, have long been a prized dog when G Malteses, originating in south central Europe, have long been prized dogs to G Originated in south central Europe, Malteses have long been a prized dog; G Originating in south central Europe, Malteses have long been prized dogs; G Malteses originate in south central Europe; they also have long been prized dogs when

23

GMAT DIAGNOSTIC 39. Currently, the town of San Hernan sends its elementary students to Carmichael Elementary School or Ira Hayes Elementary School in the neighboring town of Morton. However, San Hernan will have its own elementary school next year. Therefore, Morton will have to close either Carmichael Elementary School or Ira Hayes Elementary School next year. The argument above assumes that: G without the San Hernan students, Morton will have too few students for two schools. G the new school will not be filled to capacity for several years. G the students from San Hernan represent a small proportion of the student body of the Morton schools. G Mortons elementary schools have been overcrowded because of the extra students from San Hernan. G students will not transfer out of the district in the next twelve months. 40. The annual per capita income in State A rose from 1995 to 2000. During the same time period, the average annual household income in State A fell by more than three percent. Which of the following, if true, would best explain the discrepancy above? G The industries that were the greatest employers in State A prior to 1995 laid off much of their work force in the 1995-2000 period. G Due to divorce, a lowered birth rate, and other factors, the average number of individuals per household in State A declined from 1995 to 2000. G The software industry, which provides relatively few jobs at a relatively high payscale, grew rapidly in State A from 1995 to 2000. G During the late 1990s, the state government in State A drastically reduced property taxes. G States adjacent to State A also had simultaneous increases in per capita income and decreases in average household income in the 1995-2000 period. 41. Economist: Drastically cutting payroll costs by reducing corporate staffs will bolster corporate profits and the national economy. The remaining employees will, by necessity, operate more efficiently and work additional overtime, in the absence of now-underutilized personnel. This increase in national productivity will cause new positions to spring up with the healthier economy, providing new openings for those who were made jobless. Which of the following, if true, casts the most doubt on the economists prediction above? G If employees work longer hours, the additional hours will not be as productive as the regular hours these same employees have already worked. G Most corporations are already at the minimum numbers of employees needed to effectively maintain their operations. G Some economists predict that the national economy will substantially improve in the next two years even without drastic reductions in payroll costs. G If corporations reduce the number of employees, the average number of employees per company will decrease. G Many of the new positions in a restructured economy would be lower-paying than those lost during the restructuring.

END OF SECTION TWO

24

VERBAL

ANSWERS ON THE NEXT PAGE

25

GMAT DIAGNOSTIC

Verbal Answer Key

1. 2. 3. 4. 5. 6. 7. 8. 9. 10. 11. 12. 13. 14. 15. 16. 17. 18. 19. 20. 21. 22. 23. 24. 25. 26. 27. 28. 29. 30. 31. 32. 33. 34. 35. 36. 37. 38. 39. 40. 41.

A B B B A C B A D E A C D A A E C B D C C E B D D B B E D C C D A C A D B D A B B

26

EXPLANATIONS

EXPLANATIONS FOR SECTION 1: QUANTITATIVE

Section 1: Quantitative Answer Key


1. 2. 3. 4. 5. 6. 7. 8. 9. 10. 11. 12. 13. 14. 15. 16. 17. 18. 19. 20. 21. 22. 23. 24. 25. 26. 27. 28. 29. 30. 31. 32. 33. 34. 35. 36. 37. D B C C A B C C B A B D B D C D C E B B E C A E C E C B E B E D E A E B B

2. Correct Choice: (B) Explanation: Since we want to find the greatest amount of money that could have been received back, we want to find the smallest amount of money that could have been lost. Since losing a $20 chip is less of a loss than losing a $100 chip, we want the number of $20 chips lost to be 2 greater than the number of $100 chips lost. A total of 16 chips were lost, so we want two integers that add up to 16, and one of these integers must be 2 less than the other. You may be able to try a few possibilities very quickly and determine that the integers are 7 and 9. However, if this is not convenient, you could set up an equation. Let x be the number of $100 chips lost. Then x + 2 is the number of $20 chips lost. A total of 16 of these two types of chips were lost. So: x + (x + 2) = 16 x + x + 2 = 16 2x = 14 x=7 7 of the $100 chips were lost and 9 of the $20 dollar chips were lost. The total amount lost was ($100)(7) + ($20)(9) = $700 + $180 = $880. So the amount received back was $3,000 - $880 = $2,120. The most that the gambler could have received back was $2,120.

3. Correct Choice: (C) Explanation: Statement (1) tells us that 50 hours before the interview it was Sunday. That means the interview could have occurred any time from immediately after 2:00 p.m. on Tuesday to immediately before 2:00 a.m. on Wednesday. So we cannot answer the question and Statement (1) is insufficient. Statement (2) gives us the hours of the interview but says nothing about the day. (2) is insufficient. Taken together the statements are sufficient. From (1) we know that the interview could have happened between 2:00 am Tuesday and 2:00 am Wednesday. Statement (2) tells that the interview happened between 2:00 p.m. and 10:00 p.m., which means it happened on Tuesday. The correct answer is (C).

Explanations
1. Correct Choice: (D) Explanation: To add fractions, we find a common denominator, usually the least common multiple of the denominators in question. In this case, that would be 60. Then we convert each of the fractions into a new fraction with the common denominator. 30 + 20 + 15 + 12 + 10 87 29 9 = = 1 Here, we have = 60 60 20 20

27

GMAT DIAGNOSTIC 4. Correct Choice: (D) Explanation: To determine the area of the base of the planter, we need either the radius or the diameter of the planters base. Statement (1) gives the length of the radius of the planters base, the distance form the center of the planter to the planters edge. Knowing the planter has a radius of 9, we can determine the area of the base. (1) is sufficient. Statement (2) gives us the distance from X to W, 12 inches. We also know the length from X to the center of the planter, Z, is 15 inches. Since the line containing XW is tangent to the circular base, we know that XWZ is a right angle. We can solve for the length of ZW, the radius of the planter base. (2) is sufficient.
5 2!(5 2)!

5! 2!3!

54321 21321

= 10

Ten combinations are possible. However, since two of the members will not attend together, only 9 combinations are permissible.

Z Y W X

7. Correct Choice: (C) Explanation: If the selling price is 20% above acquisition cost we can set up an equation: 1.2A = 360,000, where A is the acquisition cost, in dollars. Solving for A, we find that A = 300,000, that is, the acquisition cost was $300,000. If the house is not sold in time the price will drop to just 5% above the acquisition cost. $300,000 increased by 5% equals $315,000, answer (C). Note: A common mistake is to just subtract 15% from the selling price, answer choice (B). Remember you must determine the original amount first before you can calculate new percentages above or below that original amount.

8. Correct Choice: (C) Explanation: First simplify the arithmetic expression under the radical symbol. + 4(121) = 300 + 484 = 784 5(60) Now we must find which answer choice equals 784. We could try squaring each answer choice to see which one squared equals 784. It might help to make a general estimate first. 202 = 400, and 302 = 900. so 784 will be between 20 and 30. We only need to check choices (B) and (C). If we solve the question this way, we will find that 282 = 784. So 784 = 28 and choice (C) is correct. Or, we could simplify 784, by factoring out perfect squares from 784. 784 = 4 196 = ( 4)(196 ) = 2196 2196 = 2 4 49 = 2(4 49) = 2(2 49) = 449 . 49 itself is a perfect square, 49 = 72, and therefore 49 = 7. So 4 49 = 4 7 = 28.

5. Correct Choice: (A) Explanation: Lets call the number of dollars that Paul has p. Since Paul has s dollars less than Jane, the number of dollars that Jane has is p + s. Together they have a total of t dollars. So: p + (p + s) = t. 2p + s = t 2p = t s ts p=
2

Therefore, Paul has

ts dollars. 2

6. Correct Choice: (B) Explanation: This question tests your knowledge of combinations. Use the combination formula. Let n and k be positive integers such that n k. Let the number of different sets of k objects that can be selected from a set of n different objects be denoted by nCk. Then n! . C = n k
k!(n k)

9. Correct Choice: (B) Explanation: To solve this, we either need to know the values of q and r or the difference between them. Statement (1)insufficient. It tells us that r = s, but doesnt give us a value for either q or r, so (1) is insufficient.

You have 5 total people, grouped into subsets of 2. So n = 5 and k = 2.

28

EXPLANATIONS Statement (2)sufficient. It tells us that tq sr + sq tr = 20. Lets try to factor tq sr + sq tr. We have that tq sr + sq tr = tq tr + sq sr = t(q r) + s(q r) = (t + s)(q r). Thus, (t + s)(q r) = 20. Since we know that s + t = 10, we have that t + s = 10. So substituting 10 for t + s in the equation (t + s)(q r) = 20, we have 20 = 2. 10(q r) = 20, and q r = 10 Statement (2) is sufficient. The correct answer is (B). Start with choice (C). If 60 taxis drive on 2nd Avenue, multiply by four to get 240 total cars on 2nd Avenue, and divide by three to find that 80 cars drive on 3rd Avenue. Too small. Try choice (D). If there are 75 taxis on 2nd Avenue, multiply by four to get 300 total cars on 2nd Avenue, and divide by three to find that 100 cars drive on 3rd Avenue. Exactly what it tells us in the stem.

10. Correct Choice: (A) Explanation: To have sufficiency, we need to know conclusively whether or not a = c or a = c. Look at statement (1). Subtracting b from both sides of the equation a + b = b c, we find that a = c. If a = c, then a = c and (1) is sufficient. Using statement (1) alone, we can answer the question with a yes. Statement (2) tells us that c a. This means that c a, that is, a c. However, we dont know whether or not a = c. So we cannot answer the question. For examples, if a = 3 and c = 10, then c a and the answer to the question is no, since However, if a = 3 and c = 3, then c a and the answer to the question is yes, since Statement (2) is insufficient. The correct answer is (A).

13. Correct Choice: (B) Explanation: Since this is a W.O.T.F question, start with answer choice (E). If x = 13, and y = 50, x ?y cannot equal 1,350. Since y = 50, y 2 = 2,500, which is much too large, so y3 will be even larger. Note that this is a rather transparent trap answer: the two numbers, placed side by side, result in 1,350. (D) gives us (2)(132)(53), or (2)(169)(125), which is also much too large. (C) results in (2)(102)(53), or (2)(100)(125), still too big. (B) equals (2)(52)(33), or (2)(25)(27) = 1,350, the correct answer. We can check (A), which results in (2)(9)(125) = 2,250, which is incorrect.

14. Correct Choice: (D) Explanation: The factors of 24 are 1, 2, 3, 4, 6, 8, 12, and 24. So, we should pick numbers and try to disprove each statement. Since its a WOTF question, start with choice (E). Choice (E): if a = 1, b = 2, and c = 3, then abc = 6, which is a factor of 24, but not a multiple of 24. Choice (D): since there are only two odd numbers in the series of factors of 24, at least one of a, b, and c must be even. Any number multiplied by an even number is even, so abc must be even. This is the correct answer.
12 1 = Choice (C) : if a = 1, b = 2, and c = 3, then 3 3

11. Correct Choice: (B) 3x , we can Explanation: Statement (1): Since 3x 1 = 3 3x rewrite Statement (1) as < 250. Multiplying both 3 sides by 3 we get 3x < 750. This does not tell us if 3x < 300 so (1) is insufficient. Statement (2): Since 3x 1 = (3x)(3), we can rewrite Statement (2) as (3x)(3) = 3x + 486 or (3x)(3) 3x = 486. We can factor out 3x to get 3x(3 1) = 486 486 3x = 2 x 3 = 243 The answer to the question in the stem is yes and (2) is sufficient.

Choice (B) : To disprove this, well look for the greatest possible numbers, so well let a = 8, b = 12, and c = 24. 8 + 12 + 24 = 44, which is greater than 40. Choice (A) : if a = 1, b = 2 and c = 4, a + b + c = 7, an odd number.

12. Correct Choice: (D) Explanation: This is a good problem to backsolve.

15. Correct Choice: (C) Explanation: Multiply out ( 8 1)(1 8 ) using FOIL.

29

GMAT DIAGNOSTIC (8 1)(1 8 ) = 18 88 (1)(1) + 18 28 64 1 = 28 9 None of the answer choices is 2 8 9. So look for a perfect square factor of 8 under the radical symbol. 9 = 24 2 9= 28 = (2)(2) 2 9 = 42 9 7(5) = 35 and b =
14 c 25

14 (25) 25

= 14. The smallest

possible value of 2a + b is 2(35) + 14 = 70 + 14 = 84.

18. Correct Choice: (E) Explanation: When you see a problem about the time it takes two entities to do a project working alone versus together use the work formula. The work formula is: ab Time Together = a+b where a and b represent the time it takes each person to complete the job on his own. Plug in the information from the problem to solve for Mikes 3b . Solve for b. time alone. Here: 2 = 3+b Cross multiply: 2(3 + b) = 3b 6 + 2b = 3b 6=b Thus, b = 6. It takes Mike working alone 6 hours.

16. Correct Choice: (D) Explanation: From the question stem, we can get the S . We need another equation with A equation A = 3 and S if we are to solve for A.
S + 15 . We now Statement (1) tells us that A + 15 = 2 have two different equations and two variables so we can solve for A. (1) is sufficient.

Statement (2) translates to S 5 = 4(A 4). Once again, we have two different equations and two variables, allowing us to solve for A. (2) is sufficient.

17. Correct Choice: (C) Explanation: Lets write each of a and b in terms of c. Look at the equation
a b a c 7 . Multiply both sides of = 5 7 c. Now look at the this equation by c. Then a = 5

19. Answer: (B) Explanation: Since this is a percent problem, use the number 100. Say the original box has a length of 100, a width of 100, and a height of 1 (since the height doesnt change, keep it small and simple). So, the volume of the box originally is 100 100 1, or 10,000. The altered box has a length of 120, a width of 80, and a height of 1. The volume of the altered box is 120 80 1, or 9,600. The question is what percent of the volume of the original rectangular box is the altered rectangular box. 9,600 96 100% = 100% = 96% That percent is
10,000 100

equation

5 . Multiply both sides by 2b. Then = 2 2a 5 2 a. = 5 7 2 2 7 14 c, b = a = ()(c) = c. 5 5 5 5 25 14 c. We see that each of a 25

2a = 5b. Divide both sides by 5, then b =


2 a and a = Since b = 5 7 c 5

Thus, a =

and b =

and b will have their smallest possible values when c has its smallest possible value. In order for a=
7 c 5

to be a positive integer, c must be a positive


14 c 25

multiple of 5 because 7 and 5 have no common factors greater than 1. In order for b = to be a

positive integer, c must be a multiple of 25 because 14 and 25 have no common factors greater than 1. Any number that is a multiple of 25 is a multiple of 5 because 25 is a multiple of 5. Now, the smallest possible value that c can have so that a (which
7 c), b (which equals equals 5 14 c), 25

20. Correct Choice: (B) Statement (1). Solve the equation x2 + 4x = 8x 35. Then x2 + 12x + 35 = 0. Use reverse FOIL to factor the left side. Then (x + 5)(x + 7) = 0. When the product of a group of factor is 0, at least one of the factors must be 0. If x + 5, then x = 5. If x + 7 = 0, then x = 7. So from statement (1), either x = 5 or x = 7. Statement (1) is insufficient. Look at statement (2). Solve the equation x2 + 3x = 7x 25. Then x2 + 10x + 25 = 0. Use reverse FOIL to factor the left side. Then (x + 5)(x + 5) = 0. So it must be that x = 5. Statement (2) leads to a single value for x. So statement (2) is sufficient. Choice (B) is correct.

and c are all

7 (25) = positive integers is 25. When c = 25, a = 5

30

EXPLANATIONS 21. Correct Choice: (E) Explanation: Use the probability formula which says that Probability = . Number of possible outcomes After the one blue marble is added to the bag, the number of blue marbles in the bag is b + 1, so the number of favorable outcomes is b + 1. After the one blue marble and five red marbles are added to the bag, the total number of marbles in the bag is r + b + 1 + 5 = r + b + 6. So the total number of possible outcomes is r + b + 6. So if a marble is selected at random, the probability b+1 , choice (E). that a blue marble is selected is
r+b+6 Number of favorable outcomes 4+s t5
5 4+ 3 20 5 17 3

15

17 3

1 = 15

17 45

23. Correct Choice: (A) Explanation: Statement (1) says that c = 8. So we have the inequality 3b < a and the equation a + b = 8. Now 3b < a. So 3b + b < a + b. So 4b < a + b. Since 4b < a + b and a + b = 8, 4b < 8. Dividing both sides of the inequality 4b < 8 by 4, we have that b < 2. Since b is a positive integer less than 2, it must be that b = 1. Statement (1) is sufficient. Choices (B), (C), and (E) can be eliminated. Statement (2) tells us that a = 7. So we have the inequality 3b < 7 and the equation 7 + b = c. Dividing both sides of the inequality 3b < 7 by 3, we 7 1 , that is, b < 2. Here, b can equal have that b <
3 3

With variables in the answer choices and in the question, we could also solve by picking numbers. Lets say that originally there were 3 red marbles, so that r = 3, and 2 blue marbles, so that b = 2. If we add 5 more red marbles and 1 more blue marble, we now have 8 red marbles, 3 blue marbles, and 11 marbles total. So, the chance of picking a blue 3 . Well check the answer choices for the marble is 11 3 when r = 3 and b = 2. ones that give a value of 11 3 when Any answer choice that does not equal 11 r = 3 and b = 2 can be eliminated.
b 2 3 = . This is not so eliminate choice (A). (A) r 3 11 2 b 2 3 so eliminate (B) = = . This is not 5 b+r 2+3 11 choice (B). b+1 2+1 3 3 = = . This is not so eliminate (C) r+5 3+5 8 11 choice (C). b+1 2+1 3 3 = = . This is not so (D) r+b+5 3+2+5 10 11 eliminate choice (D).

either 1 or 2. Statement (2) is insufficient. Choice (A) is correct.

24. Correct Choice: (E) Explanation: Start with answer choice (E) since this is a W.O.T.F. question. y 2x = 1 can be rewritten as y = 2x + 1. This line has a y-intercept of 1 and a positive slope of 2. Since none of the lined bounding the shared shape has a positive slope, (E) cannot be one of the boundary lines. So choice (E) is correct. Just to be sure, lets look at the other 4 answer choices. One of the boundary lines is the portion of the xaxis, which goes from the origin at (0, 0) to the point (1, 0) on the x-axis. The y-coordinate of every point on this line is 0. So the equation of this boundary line is . Eliminate choice (A). Another boundary line of the shaded region is parallel to the x-axis. This line goes through the 1 , 1). So the y-coordinate of every points (0, 1) and ( 2 point on this line is 1. Thus, the equation of this boundary line is . Eliminate choice (B). Another boundary line is the portion of the y-axis which goes from the origin at (0, 0) to the point (0, 1) on the y-axis. The x-coordinate of every point on this line is 0. So the equation of this boundary line is . Eliminate choice (C).

Now that all 4 incorrect answer choices have been eliminated, we know that choice (E) must be correct. Just to be sure, lets check choice (E). b+1 2+1 3 = = . Choice (E) does work. (E)
r+b+6 3+2+6 11

22. Correct Choice: (C) Explanation: Find the values of s and t and then plug 4+s . these values into the expression
t5 5 = 3, we have that 5 = 3s, and From the equation s 5 then s = . From the equation t = 4, we have that t 3 5 = 20. Now substitute the values of s and t into the expression.

31

GMAT DIAGNOSTIC The fourth boundary line is the line that contains the 1 , 1), and (1, 0). Lets work with the points (0, 2), ( 2 points (0, 2) and (1, 0). If the equation of this written in the slope-intercept form , then for the slope m we have 02 2 = = 2. m= 10 1 The equation of the line is y = 2x + b. Since the point (0, 2) is on the line with the equation y = 2x + b, substitute 0 for x and 2 for y into the equation y = 2x + b. Then 2 = 2(0) + b, 2 = 0 + b, and b = 2. The equation of this boundary line is y = 2x + 2. Eliminate choice (D). Again, choice (E) is correct. Statement (2) gives the distance between Stations N and P, but we dont know where Station O lies along that distance. (2) is insufficient. Even with Statement (1) and (2) together, we cannot determine the distance between N and O. As the diagrams below show, N can be closer to M or closer to O. Therefore the distance between N and O cannot be determined.

N 15 12

25. Correct Choice: (C) Explanation: This is a yes/no question. We must jump right to the statements, remembering that if it turns out that it is always true that r 4, the answer is NO, which is sufficient. Statement (1) gives us a quadratic equation. Factoring, the left side of the equation r 2 3r 10 = 0, we get the equation (r 5)(r + 2) = 0. When the product of a group of factors is 0, at least one of the factors is 0. So either r 5 = 0 or r + 2 = 0. If r 5 = 0, then r = 5. If r + 2 = 0, then r = 2. So r could equal either 5 or 2. If r = 5, then r > 4 and the answer to the question is YES. However, if r = 2, then it is not true that r > 4 and the answer to the question is NO. When our answer is maybe, the statement is insufficient. Eliminate answer choices (A) and (D). Statement (2) gives us the product of three factors and says that the product of these three factors is 0. When the product of a group of factors is 0, at least one of the factors is 0. So if r(r 5)(r 6) = 0, at least one of the factors r, r 5, and r 6 must be 0. When the factor r is 0, we simply have r = 0. If r 5 = 0, then r = 5. If r - 6 = 0, then r = 6. So r equals 0, 5, or 6. Is r > 4 ? Maybe; zero is less than four, but five and six are greater than four. Statement (2) is insufficient; eliminate answer choice (B). When both statements are taken together, there is only one value of r that is a solution to both equations: r = 5. Is r > 4 ? Yes. Both statements together are sufficient; the correct answer is (C). 26. Correct Choice: (E) Explanation: Statement (1) gives us the distance between M and O, but we dont know where Station N lies along that distance. (1) is insufficient.

M 15

O 12

27. Correct Choice: (C) Explanation: This is a good question to solve by picking numbers. When picking numbers, all 4 incorrect answer choices must be eliminated. Choice (E): Say a = 5 and b = 3. (52)(32) (5)(3) = (25)(9) 15 = 225 15 = 210. 210 is not prime. Eliminate choice (E).
1 = (5)(3) = Choice (D): Same numbers: (52)(32) (5)(3) 15. Not prime. Eliminate choice (D). 1 1 ] = (75)() Choice (C): Same numbers: [(52)(3)][ (5)(3) 15 = 5. This is prime. This choice is possibly correct. Choice (B): Same numbers: a b = 5 3 = 2. This is prime. This choice is possibly correct. Choice (A): a = 5 and b = 3, then a + b = 5 + 3 = 8. Not prime. Eliminate choice (A).

We are down to choices (B) and (C). We must go back and pick another set of values for a and b that are consistent with the question stem. That is, lets pick another set of values for a and b that are both prime. This time let a = 7 and b = 3. Choice (B): a b = 7 3 = 4. Not prime. Eliminate choice (B). Now that all 4 incorrect choices have been eliminated, we know that choice (C) must be correct. Lets check that choice (C) is prime when a = 7 and b = 3.

32

EXPLANATIONS
1 1 ) = (49)(3)() = Choice (C): (723)( (7)(3) 21 is prime. 147 21

= 7. This

1 ) must be We can prove algebraically that (a 2b)( ab 1 ) = prime. We have that (a2b)( ab aa b ab

= a. The

question says that a must be prime, so we have


1 ) must be prime. shown that (a 2b)( ab

cost, in dollars, of using car service Y is 10 + 1.28d. The two costs are equal. So 14 + 1.2d = 10 + 1.28d. Now solve this equation for d. Subtract 10 from both sides. Then 4 + 1.2d = 1.28d. Subtract 1.2d from both sides. Then 4 = 0.08d. 4 = d Divide both sides by 0.08. Then 0.08 400 4 4 100 = 50. Then d = = = 8 0.08 0.08 100 Thus, d = 50.

28. Correct Choice: (B) Explanation: The sum of the numbers if the vertical column is 12 + 14 + b. The sum of the numbers in the horizontal row is a + b + 14. The two sums are equal. So 12 + 14 + b = a + b + 14. Subtracting b from both sides, we get 26 = a + 14. Subtracting 14 from both sides, we get 12 = a.

31. Correct Choice: (E) Explanation: The stem tells us that a is a two digit integer. It can be any number from 10 through 99 inclusive. Statement (1) limits the set of integers that can be a. However a could be 54, 45, or several other integers. (1) is not sufficient. Statement (2) is also not sufficient. There are several numbers that are two digits and are multiples of 3. a could be 27 or 30. (2) is not sufficient. Together, the statements are still not sufficient. For example, both 45 and 27 have digits that add up to 9 and both 45 and 27 are divisible by 3. The correct answer is (E).

29. Correct Choice: (E) Explanation: Statement (1) gives us the length of one side of triangle ABC. We cannot figure out the length of side AB. (1) is insufficient. Statement (2) gives us the length of one side of triangle ABC. We cannot figure out the length of AB. (2) is insufficient. When both statements are taken together, we know the length of two sides of the triangle. We know that the length of a side of a triangle must be less than the sum of the lengths of the other two sides and greater than the positive difference of the other two sides. So we know that 10 7 < AB < 10 + 7, that is, 3 < AB < 17. We can have AB = 7, which is less than 12. In this case, the answer to the question is YES. We could also have AB = 16 which is greater than 12. In this case, the answer to the question is NO. There is not enough information to answer the question in the stem when the statements are taken together. The answer is (E).

32. Correct Choice: (E) Explanation: We need to work with the equation to isolate x. Start by multiplying both sides by (x - y), which gives us x + y = 4(x y). Now multiply out the right side so that we wont have to work with parentheses. Then x + y = 4x 4y. Gather the xs and ys to get 3x = 5y. Then divide both sides by 3 5 y, answer choice (E). to get x =
3

30. Correct Choice: (B) Explanation: Let d be the distance, in miles, at which the cost of using the two car services is the same. The initial cost for using car service X is 14 dollars. The cost per mile when using car service X is 1.2. So not including the initial cost, the cost of dollars d using car service X for d miles is 1.2 mile miles, or 1.2d dollars. The total cost, in dollars, of using car service X is 14 + 1.2d. Similarly, the total

33. Correct Choice: (E) Explanation: Try picking numbers to determine the sufficiency of the statements. For statement (1), try both a positive and a negative number to evaluate the stem. (53)2 56 2 If x = y = 2, then 4 = 5 2 2 =
(5 ) 5

If x = y = 2, then

(53)2 2 = (52)

1 3 2 (5 ) 1 2 2 (5 )

1 56 1 54

54 56

1 . 52

Clearly, there is not a single value for

(53)x 2 y (5 )

when x =

33

GMAT DIAGNOSTIC y so statement (1) is insufficient. Statement (2) states only that x is positive. So if x = y = 3 then = the value
(53)3 59 2 = 53. Comparing this value to 3 = (5 ) 56 (53)x 52 of y when x = y = 2, there is not one (52)

single value for the expression in the stem and (2) is insufficient. Now lets consider the statements together. When x = y = 2, both statements are true and we (53)x 2 have seen that the value of 2 y is 5 .
(5 )

Probability = . Number of possible outcomes That is, we can find the probability by calculating the number of desired outcomes divided by the number of possible outcomes. There are 288 desired outcomes, which are the number of pages with text. There are 300 possible outcomes because there are a total of 300 pages. So the probability that a single 288 which can be copy is missing a page of text is 300 24 reduced to .
25

Number of favorable outcomes

Choice (E) is correct.

When x = y = 3, both statements are true and we (53)x 3 have seen that the value of 2 y is 5 .
(5 )

When both statements are taken together, there is (53)x no single value for 2 y and the correct answer is (E).
(5 )

36. Answer choice: (B) Explanation: First, add the two figures. 18 hours and 20 minutes + 6 hours and 40 minutes = 24 hours and 60 minutes = 25 hours. Now find the total number of hours in a week. 7 days/week 24 25 of hours/day = 168 hours/week. So 25 hours is 168 a week. Choice (B) is correct.

34. Correct Choice: (A) Explanation: The stem gives us percents, but no actual numbers. If we knew the total number of orchestra members, we could determine the number of female string players. Statement (1) lets us set up an equation. Since 120 members are string players and 40% of the members are string players, we can write 0.4t = 120. Solving for t (the total number of orchestra 120 120 10 1,200 = = = 300. members) we get t = 0.4 0.4 10 4 We can find 28% of 300 to determine the number of female string players. Statement (1) is sufficient. Eliminate choices (B), (C), and (E). Statement (2) gives us another percentage, but we still do not have any actual numbers. We cannot solve for the number of female string players so statement (2) is insufficient. Choice (A) is correct.

37. Correct Choice: (B) Explanation: In order for m + 4 to be even, m needs to be even. The stem is essentially asking, is m even? Statement (1): Pick different values for m. If m is odd, then 2m + 1 is odd. For example, if m = 1, then 2m + 1 = 2(1) + 1 = 2 + 1 = 3, which is odd. However, if m is even, 2m + 1 is also odd. For example, if m = 2, then 2m + 1 = 2(2) + 1 = 4 + 1 = 5, which is odd. We cannot determine if m is odd or even so (1) is insufficient. Eliminate choices (A) and (D). Statement (2): This statement says that 2m + 4 is a multiple of 4. Now 4 is also a multiple of 4. When two integers are multiples of 4, then the first integer minus the second integer is a multiple of 4. That is, if a is a multiple of 4 and b is a multiple of 4, then a b is a multiple of 4. For example, 20 is a multiple of 4 (20 = 5 4) and 12 is a multiple of 4 (12 = 3 4). Then 20 12 = 8 and 8 is a multiple of 4 (8 = 2 4). Therefore, since (2m + 4) is a multiple of 4 and 4 is a multiple of 4, (2m + 4) 4 = 2m + 4 4 = 2m is a multiple of 4. Now 2m is a multiple of 4 means that 2m contains the prime factors of 4. The prime factorization of 4 is 2 2. Only one of the two prime factors 2 (of 4 = 2 2) is contained in the factor 2 of 2m. Therefore the other prime factor 2 (of 4 = 2 2) must be contained in the prime factorization of m. That is, m must be a multiple of 2. That is, m

35. Correct Choice: (E) Explanation: There are two types of pages among the 300 pages. There are 288 pages that have text and 12 pages that are blank, that is, do not have text. We can think of this situation in this way. We are selecting one page at random and we want to know the probability that the page has text. The probability formula says that

34

EXPLANATIONS must be even. Now m is even and 4 is even. So m + 4 must be even. Statement (2) is sufficient. You can also pick numbers in order to consider statement (2). If m = 0, 2m + 4 = 2(0) + 4 = 0 + 4 = 4 is a multiple of 4. If m = 2, 2m + 4 = 2(2) + 4 = 4 + 4 = 8 is a multiple of 4. In fact, any positive even integer value for m makes 2m + 4 a multiple of 4. Conversely, if m = 3, 2m + 4 = 2(3) + 4 = 6 + 4 = 10 is not a multiple of 4. If m = 5, 2m + 4 = 2(5) + 4 = 10 + 4 = 14 is not a multiple of 4. If m equals any odd number, 2m + 4 is not a multiple of 4. Therefore m is even. We know that 4 is even. Therefore m + 4 is even and statement (2) is sufficient.

35

GMAT DIAGNOSTIC

EXPLANATIONS FOR SECTION 2: VERBAL

Section 2: Verbal Answer Key


1. 2. 3. 4. 5. 6. 7. 8. 9. 10. 11. 12. 13. 14. 15. 16. 17. 18. 19. 20. 21. 22. 23. 24. 25. 26. 27. 28. 29. 30. 31. 32. 33. 34. 35. 36. 37. 38. 39. 40. 41. A B B B A C B A D E A C D A A E C B D C C E B D D B B E D C C D A C A D B D A B B

Explanations
1. Correct Choice: (A) The sentence is correct as written. Lets see why the other answer choices are incorrect. Choice (B) leaves us with two independent sentences (the workdeveloped and the methodcontinued) joined incorrectly by a comma (a comma splice). Choice (C) is incorrect because the work (now the subject of an independent first clause) does not have a corresponding action (verb). Choice (D) is incorrect because it uses the wrong form of the verb develop (which is used as an adjective in this sentence). Choice (E) changes the meaning of the original sentence, which told us that the field began with Schmidts work. As the sentence is now, we are told that the field continued into the 1980s with Schmidts work. Remember that the original meaning of the sentence must be maintained. 2. Correct Choice: (B) All items in a list must be expressed in parallel form. Two items in the original version are gerunds (treating..., relieving...), but the third is not. The simplest correction is to make the third item a gerund as well, and only choice (B) correctly does this (dulling pain). Choice (C) uses dulling too, but needlessly and erroneously changes one of the original terms (treating to the treatment of). Choices (D) and (E) use the singular construction which includes, which does not agree with the plural applications. 3. Correct Choice: (B) The physicians recommend that high-homocysteine people consume more folate, so the best answer choice will be one that shows that more folate means less homocysteine. Choice (B) does just that. Choice (A) weakens the recommendation (by noting other possible deep vein thrombosis risk factors) and tells us nothing about folates relationship to homocysteine. Choice (C) tells us that the inverse of the physicians recommendation has other negative effects, but that doesnt mean that taking more folate will reduce homocysteine levels. Choice (D) simply adds to the evidence that deep vein thrombosis is life-threatening. Choice (E) says that folate seems to have no side effects; this doesnt provide any evidence that the treatment will have the desired effect.

36

EXPLANATIONS Topic: Depression and dementia among the elderly Scope: The differences between depression and dementia Purpose: To explain the difficulty of distinguishing between depression and dementia in the elderly. Paragraph 1 explains the overlap in symptoms between the two disorders. Paragraphs 2 and 3 provide specific examples of differences in symptoms. 4. Correct Choice: (B) The author wants to explain why it is hard to distinguish between depression and dementia in elderly people, so choice (B) is correct. Choice (A) is incorrect because the problems arising from incorrect diagnoses are implied, but never addressed explicitly. Choice (C) is incorrect because the author is not primarily trying to compare the reliability of different means of differentiating between the two disorders. Choice (D) is outside the scope of the passage. Choice (E) is tempting; the author mentions differences in cognitive performance in paragraph 3, but this is not the primary focus. 5. Correct Choice: (A) Statement I is the only one that is correct, so choice (A) is the answer. We are told in the first paragraph that it is difficult to distinguish dementia from depression because the symptoms overlap. Statement II is incorrect because the author never discusses and tendency of those with one disorder to succumb to the other, just the symptoms of each. Statement III is wrong because, while organic causes are implied for dementia (at the end of the first paragraph), they are never associated with depression. 6. Correct Choice: (C) To answer this question, we need to look at the relevant sentence and its context. The author uses olfactory problems as an example of one way that dementia patients differ from depressed ones; thus choice (C) is the answer. Choice (A) is nearly the opposite of the correct answer and a misused detail from the passage; this statement is part of the authors overall reasoning, but not relevant to this question. Choice (B) suggests a possible reason why dementia patients lose their sense of smell and depressed individuals do not; although it may be a reasonable explanation, it isnt in the passage, and it is not the reason why the author discusses olfactory problems. Choice (D) is wrong because the author never discusses the breadth or quantity of symptoms of either depression or dementia. Choice (E) is outside the scope of the passage, which is about the elderly who suffer from dementia or depression, not about the elderly in general. 7. Correct Choice: (B) This is a scattered detail question; we should eliminate the choices we can find support for in the passage. According to the last paragraph, choice (B) is a symptom common to depression and dementia (though to varying degrees). All of the other choices are ways to distinguish between the two. Choice (A) refers to the olfactory problems discussed in paragraph 2. Choice (C) corresponds to the information about response biases presented in paragraph 3. Choice (D) is discussed in the second paragraph. Choice (E) can be eliminated as well, because the author says that antidepressants will generally improve the cognitive dysfunction associated with depression but will have minimal effect on organic degeneration (lines 1821). 8. Correct Choice: (A) In order to properly conclude, or infer, something new, we often must combine pieces of evidence from the stimulus to draw a conclusion. Choice (A) properly combines the information that paleontologists attempt to reconstruct early Cambrian ecosystems with the fact that only under exceptional circumstances do animals with soft body parts become fossilized. In addition, the tone of (A) reflects the moderate language of the stimulus (difficulty ... most ... may ... exceptional circumstances). Choice (B) contradicts information in the passage; we are told in the first sentence that: most animal specieslacked hardened body parts. Choices (C) and (D) erroneously translates typically preserved hard parts and exceptional circumstances under which soft-parted animals are preserved into the extreme assertions that no traces are found of animals lacking those parts. Choice (E) is also too extreme: difficulty becomes impossible to referring to the chances of accurate reconstruction. 9. Correct Choice: (D) Since the change resulted in a loss of productivity, we can infer that the part-time employees were more productive than the freelancers who were hired in their place. A reasonable solution, then, is to rehire those productive part-timers as freelancers. Theyll still be the same productive employees. Theyll simply be paid differently. Choice (A) sounds reasonable until we consider that we dont know the number of remaining part-time employees. There may be so few compared to former numbers that increasing the hours wont significantly

37

GMAT DIAGNOSTIC affect productivity. Choices (B) and (C) seem like good ideas, but neither option is indicated by the information in the passage. There is no evidence that the managers are not already sufficiently motivated or that the advice of efficiency experts is needed. Choice (E) might be efficient, but having fewer total employees is certainly not likely to increase productivity. 10. Correct Choice: (E) A list beginning with from must include to at some point. The necessity of to eliminates choices (A) and (C). Choice (B) violates parallelism, by using verbs (cleaning, increasing) as the second and third elements in a list that started with a noun phrase. Choice (D) includes to but the parallelism is broken by the third element, increasing. Only choice (E) includes the preposition to and sustains the parallelism. 11. Correct Choice: (A) Answer choice (A) uses the correct plural verb form are with the plural subject items. At is the correct preposition with verb aimed. Choice (B) uses the incorrect preposition to; choice (C) also uses to and adds an apostrophe to consumers, unnecessarily making it possessive; choice (D) makes the verb singular, which no longer agrees with the plural subject; and choice (E) changes both the verb to the singular and the preposition to to. 12. Correct Choice: (C) Explanation: Lists must be expressed in parallel form. In this case, either repeat the helping verb is before every term in the list, or place it only before the first term. Choice (A) thus adds an is where none is needed. Choices (B) and (E) add an unnecessary now. Choice (D) completely muddies the expression by adding along with being.... Only choice (C) correctly lists Johns old friends current attributes in the most economical and parallel way. 13. Correct Choice: (D) The conclusion is that production of the drug must lead to the spiders extinction. The evidence is that it can only be obtained from the webs of a species that is endangered. So the assumption is that collecting the webs will kill the spiders. But if, as choice (D) suggests, the spiders move on to a new web everyday, scientists might be able to collect the old webs without endangering the species. Choices (A) and (C) both provide more reasons that there will be a very wide demand for the spiders and their webs, and thus further support the conclusion. Choices (B) and (E) suggest that the anesthetic will be difficult and expensive to manufacture, but cannot guarantee that demand will not provide motivation to overcome these obstacles. Topic: Bias in journalism Scope: How certain biases are inherent in the nature of journalism Purpose: To discuss the biases inherent in journalism Paragraph 1 deals with the differences between conventional biases and two types of bias inherent in journalism: structural and political assumption. Paragraph 2 shows how the need to sell papers and the nature of the audience cause biases. Paragraph 3 shows that attitudes toward government and politics, as influenced by the biases discussed in paragraph 2, are the source of biases. Paragraph 4 gives an example of how these biases make analysis difficult. 14. Correct Choice: (A) Choice (A) is the credited choice. The business decisions first discussed in paragraph 2 are at the core of journalistic bias, affecting decisions made as described in paragraph 3. Choice (B) is a distortion of facts relevant only to paragraph 2. Choice (C) distorts the discussion found only in paragraph 3. Choice (D) refers to paragraph 4 only. Choice (E) is outside the scope of this passage, which never deals with ways to avoid these biases; in fact, the authors main idea is that these biases cant be avoided. 15. Correct Choice: (A) Choice (A) comes closest to the authors ideas. Structural biases are the subject of paragraph 2 (in contrast to the political assumptions also introduced in paragraph 1 and discussed in detail in paragraph 3). Choice (B) appears at the end of paragraph 2 as one example of this type of biasnot as its primary cause. Choice (C) infers too much from the passages references to an unsophisticated audience. Choice (D) is not discussed in this passage; the competition is alluded to, but we arent told its growing. Choice (E) is outside the scope of this passage, which says in paragraph 1 that this type of conventional bias is less relevant than the types discussed here. 16. Correct Choice: (E) Progressivist ideas are mentioned in paragraph 3, but we must be careful to note the context of this reference. We are told in the first paragraph that simple bias for a particular party isnt at the root of journalisms biases, and in the second that what does shape journalism is a preference for a good, simple story. So the author isnt saying that journalists are likely to adhere to Progressivist (or

38

EXPLANATIONS any other particular partys) dictates, as statement I says. Statement II is outside the scope; whether or not any biases are masked successfully is never discussed here. Statement III effectively restates the nature of the structural biases discussed in the passage, so choice (E) is the credited choice. 17. Correct Choice: (C) Generally, the author does not offer any opinions about politics or campaign finance in this passage, only about the nature of journalism, so only choice (C) can be correct. In the second half of the third paragraph, PACs are called the embodiments of vested, selfish interests in connection with campaign finance, but dont confuse that with a general statement about campaign finance itself. Choices (A) and (B) suggest the views of the Progressive muckrakers in paragraph 3, not of the author. Choice (D) seems to be a distorted reference to irrelevant language in the first paragraph: There is no singling out of Democrats or Republicans. Choice (E) is another distortion; it might be correct if the question were about journalism, rather than about campaign finance. 18. Correct Choice: (B) We must look at the last paragraph, where this conundrum appears. Reading for context shows us that the author doesnt know whether publishing decisions are made because journalistic biases have preconditioned the audience to accept certain stories, or whether they simply believe they are printing the truth: Choice (B) captures the essence of the problem. Choice (A) is an extreme misinterpretation of paragraph 3, not relevant to the question. Choice (C) misstates the authors points; these two forms of bias are never described as conflicting. Choice (D) joins ideas from paragraphs 2 (readers preferences) and 3 (Progressivist ideas) into a distorted version of the passages ideas. Choice (E) is a detail from paragraph 2, not relevant to the conundrum in paragraph 4. 19. Correct Choice: (D) Consider the authors overall point: that journalisms need for a good, simple story affects all of its decisions. Now look at the references to PACs: they are offered as the embodiment of the Progressive views that favor a good guys/bad guys interpretation. Thus choice (D), simplistic, is correct. Choices (A), (C), and (E) are outside the scope of the passage; this author doesnt offer any opinion about whether the evaluation, or the information it is based on, is correct or not. The passage only says its an easy target. Choice (B) expresses the opposite of what the author thinks. 20. Correct Choice: (C) To cast doubt on the arguments conclusion, we should try to undermine the assumption(s) linking it to the evidence. In this case, the author seems to be assuming that the only reason for the stores success is its publicity campaign. Choice (C) undermines this assumption, by suggesting that Movie Tapes has succeeded because it sells movies not available in DVD format. Choice (A) would make DVDs more attractive, but would not provide an alternate explanation (other than the publicity campaign) for Movie Tapes sales success. Thus it is incorrect. Choices (C) and (D) suggest that the school system and the library, respectively, are potential buyers of videocassettes, but we dont know that either of them alone could account for Movie Tapes tremendous sales, or that they bought only from Movie Tapes. Choice (E) neither strengthens nor weakens the argument, and so is irrelevant to this argument. 21. Correct Choice: (C) This sentence is contrasting the differences between collaborative law and mediation. In collaborative law the outcome is decided by the lawyers and litigants. In mediation, the outcome is decided by a mediator. Under the rules of parallelism, the two items compared must be in the same grammatical form. Only choices (C), (D) and (E) supply the word by required for parallelism. However, choice (D) creates parallelism, but loses the element of contrast found in the original. Choice (E) is wordy and awkward, and decided by lawyers and litigants is not parallel to being done by a mediator. 22. Correct Choice: (E) The argument concludes that if a two-year-old car is resold for only 10,000 USD, it has experienced greater than average depreciation. But this assumes that its original price was within the average range defined in the evidence. It is conceivable that its original price was well below average; if so, then even with average depreciation it could be resold for only 10,000 US dollars after two years. So choice (E) is the best answer. Choice (A) is a true statement, but irrelevant to the argument: the only measure we are concerned with is resale value. Choices (B) and (C) describe specific cases of cars that are outside the average price ranges, but they do not point to general flaws in the argument. Choice (D) brings up the issue of poor design, which is totally outside the scope of the stimulus.

39

GMAT DIAGNOSTIC 23. Correct Choice: (B) The conclusion is set off by the words The company is certain that . . . The rest of the paragraph is the evidence. The assumption that links evidence to conclusion is that the greater cost of the new furniture will make consumers think that it is of greater quality. At no point does the paragraph state that the new product line will actually be of greater quality. (B) sums up this assumption. Choice (A) is too extreme. Although the argument assumes that consumers think that more-expensive furniture has more quality, theres no reason to think that consumers believe that there is an exact one-toone correspondence as rigid as (A) claims. Choice (C) says consumers favor one characteristic (high quality) over another (low cost), but we are given no reason to believe this. The company is having problems because consumers dont feel that its lowcost line has a lot of quality, from which it can be inferred that consumers want lost-cost chairs and tables to have at least some qualitybut theres nothing in the paragraph about one being more important than the other. Choice (D) may or may not be true, but its not an assumption in this argument. The company is going to charge more in order to give an impression of quality, not simply to have a greater profit per item. Choice (E) may be tempting, because it appears to connect with concerns about quality, but theres nothing in the text to support that idea that consumers care about quality because they want their furniture to last for a long time. 24. Correct Choice: (D) The conclusion is that the greatest chance for life to exist is outside our solar system because we know that there are earth-like planets in other solar systems. To link evidence and conclusion, we must assume that there is a connection between the presence of earth-like conditions and an increased chance for the existence of life. So choice (D) is correct. Choice (A) is out of scope, since were concerned only with whether life exists, not what form it takes. Choices (B) and (C) are too extreme (note the strong wording: cannot exist and will always). Choice (E) is out of scope, since were not concerned with detecting life, but with whether it exists. 25. Correct Choice: (D) This sentence sounds strange and does not make complete sense. It is unclear whether a comparison is being made between the rotation speed of the Earths inner and outer cores OR if the inner core is being rotated BY the outer core. Either way, there is a problem here. Since the meaning of the sentence must be clarified, we must go to the answers to see what the intended meaning is. We know immediately that we can eliminate choice (A), since the sentence is unclear as written. Choice (D) creates a clear, concise sentence whose subject and verb and makes sense. Choice (B) creates new errors. The pronoun it is too far from its antecedent, the earths inner core; the construction is awkward; and there is no verb for what sounds like the primary subject of the sentence, The Earths inner core, a ball of solid iron. Choice (C) unnecessarily places the pronoun it immediately after its antecedent. Choice (E) is wordy and awkward. 26. Correct Choice: (B) The argument states that residents should enroll in the plan in order to save money. If, as choice (B) suggests, residents would make more money by investing the equivalent costs now and paying the expenses themselves when the time comes, then the argument is wrong: it would not be financially prudent to take part in the program. Since the program covers participants at any staterun facility, residents should be able to move anywhere within the state, and still fall under the programs guidelines, and choice (A) is wrong. Choices (C) and (D) both strengthen the argument by stating that it will be less expensive to pay for the procedures now. Even if some procedures will not be covered, the fact that many others are covered still suggests that the plan is a good option; so choice (E) is also incorrect. 27. Correct Choice: (B) The driver completed both actions, waiting and deciding, in the past, so choice (A)s has waited which implies that hes still waiting is wrong. The other problem here is that, while its acceptable in conversation to say the driver waited before deciding his client wasnt going to show up, GMAT English requires a that to introduce the independent clause his client was not going to show up. Choice (B) corrects both of these mistakes. Choice (E) corrects the first mistake but not the second. Choice (C) corrects the second but not the first; moreover, (C) also errs by needlessly changing deciding to he decided. Choice (D) incorrectly uses the past participle form, was waiting, and neglects to put a that after deciding. 28. Correct Choice: (E) The question is: what is the antecedent of who? As written, the antecedent of who is two, but, there is a group of pandas, and two of the pandas live in the National Zoo. One correct construction for

40

EXPLANATIONS identifying a subgroup from a larger group is to use (number) of (category). To replicate this construction with a relative pronoun, the antecedent of the relative pronoun must be pandas in captivity; thus, two of which. Choice (A) is almost right, but two of who is ungrammatical (two of whom would work). Choice (B)s use of and suggests that the pandas at the National Zoo are being considered in addition to pandas in captivity, which makes no sense; they are in fact included among the captive pandas. Choices (C) and (D) are wordy and ungrammatical. Choice (E) correctly uses an objective relative pronoun as the object of the preposition of. 29. Correct Choice: (D) This sentence has both an error in subject-verb agreement and an incorrect use of an idiom. We need the subject and verb to agree in number. Either a tomato contains or tomatoes contain. Answer choices (A) and (B) can be eliminated since they both have subject-verb agreement mistakes. Choice (C) contains a parallelism mistake: As well as Vitamin C and Vitamin A is not parallel to a tomato. Choices (D) and (E) both correct the agreement mistake, but only (D) correctly uses the not onlybut also construction. 30. Correct Choice: (C) If the school is losing hotel and restaurant students because the price was raised for degree-credit courses, one solution, as outlined by (C), is to simply lower the price for those course that hotel and restaurant professionals are likely to take. Choices (A), (B), and (E) all might make classes more attractive for the target audience, but none of these choices address the main problemthe increased price. Choice (D) might make the noncredit courses less attractive, but it wont make the credit courses any more attractive. Topic: the computer chip market Scope: factors affecting the price of computer chips Purpose: to show that computer chips are susceptible to hog cycles Paragraph 1 gives a brief history of the chip market Paragraph 2 describes how hog cycles work in the chip industry 31. Correct Choice: (C) This question asks us to determine the authors purpose. The author discusses the drop in computer chip prices, and explains this by comparing the semiconductor business to another industry farming. We need an answer that matches this idea in both scope and tone: (C) does. Choice (A) begins with the word argue, but the author is not trying to argue a point. Similarly, the author is not trying to contrast the problems of different industries, as in choice (B). Choice (D) is too extreme: the author does not try to prove that wild swings are inevitable. Choice (E) is wrong in scope; only a single problem, shared by two specific industries, is discussed. 32. Correct Choice: (D) This question asks us to describe the structure of the passage. To answer, we need a sense of the purpose of each paragraph, and how each relates to the main idea. The author begins the passage by introducing a problem in one industry, and then explains this problem via another industry that experiences a similar problem. This is essentially what choice (D) tells us, so it is the best answer. Choice (A) is incomplete because it fails to mention that another industry is used to explain the situation. Choice (C) goes beyond the scope of the passage, which does not discuss how to control the problem. Choice (B) is a distortion, since the passage does not discuss similar industries. Choice (E) is another distortion, because the passage does not focus on related problems in two industries, and it does not attempt to contrast them. Most of the passage focuses on the semiconductor industry, with farming mentioned only to shed light on problems within the computer chip market. 33. Correct Choice: (A) This question asks the purpose of a detail. Our job is to figure out the context in which the cited statement was used. This sentence occurs right after the author writes that the book-to-bill ratio for semi-conductor manufacturers fell to around 0.9. In other words, the author uses the detail (for every $100...) to clarify what is meant by a drop in the book-to-bill ratio. This is exactly what choice A states. Choice (B) misuses details from the passage; the fall in chip prices is discussed elsewhere in the passage, but isnt directly relevant here. Choices (C), (D), and (E) are misused details from paragraph 2, not the reasons for this example. Note the importance of looking back at the context, especially for detail questions. 34. Correct Choice: (C) This question asks us to make an inference. The correct answer to an inference question will never be too far from what is stated in the passage. We are told, Producers tend to reduce output when demandfalls, but that future production isnt as smoothly tied to immediate changes in demand. It is only certain that rises in demand lead to shortages

41

GMAT DIAGNOSTIC and high prices. Only choice (C) will necessarily follow. Choice (A) contradicts what the passage says: that the chip industry cannot turn on a dime and rapidly produce more chips. Choice (B) cant be inferred: the industry would probably, but not necessarily, respond by building new fabrication plants; but the industry may foresee reasons why the current boom in demand will not last. Dont read into the passage information or opinions you have from other sources. Choice (D) is the opposite of what we would expect. Were told that when demand fell, book-to-bill ratio fell: for each $100 worth of chips produced, only $90 worth of new orders were received; we would assume it would rise if demand rises. Choice (E) may seem a likely sequence of events, but it isnt necessary. Its projecting too far into the future, with too many variables that might affect the outcome. 35. Correct Choice: (A) Sentences like this one, with its complex introductory phrase, can be intimidating, but theres nothing wrong with the original version: the introductory phrases correctly modify the sentences subject the Baby Boomers. Choice (B)changing having seen to seeingalters the time sequence of the original: the Baby Boomers children are not presently going out into the world; they have already gone out into the world. Choice (C) needlessly complicates each modifying phrase with improper inversion of the word order and a switch to the passive in the second. Choices (D) and (E) inappropriately introduce the idea of contrast. The author is actually saying that, because they have taken care of these other things, the Baby Boomers now have more money to spend than they previously did, not although or despite. 36. Correct Choice: (D) The sentence begins with a subordinate clause containing an unspecified pronoun it. The pronouns antecedent James Deans career must directly follow this subordinate clause; the most precise choice is therefore (D). Choice (A), incorrectly implies that the it refers to James Dean himself, but Dean would be a he, not an it and he doesnt consist of three films. Because of its last two words, choice (B) makes no sense when plugged into the sentence: had that generated? Choices (C) and (E) make for correct pronoun reference, but are awkward and passive. 37. Correct Choice: (B) If the deaths of the SleepSoon consumers were known to be unrelated to the drug (perhaps they were all diagnosed with cancer), then the deaths would certainly not be reason for the health agency to deny approval. So choice (B) is the best answer. Choices (A) and (D) provide more evidence for not approving the drug, so these answer choices certainly dont support the health agencys decision. Choice (C) mentions new regulations, but we dont know whether it would be more be more or less difficult for SleepSoon to receive approval under these guidelines. Choice (E) cannot have any bearing on the approval of the drug by the health agency. 38. Correct Choice: (D) This question is almost impossible to answer without first reading all of the answer choices. The adverbial conjunction when seems to set up an impossible sequence of events; the correct answer should alter or even dispose of when altogether. Choice (D) replaces when with a semicolon to express several ideas in a clear, concise way. It also correctly uses the plural dogs to describe Malteses. Choice (B) begins well, streamlining the relative clause (originating from), but, in altering when to to, it makes Egyptians have the impossible double function of object of to and subject of are thought. Choice (C) needlessly uses the past participle originated to modify Malteses instead of the present participle (originating). Choice (E) also uses a semicolon, but the division of ideas is awkward and the when problem is uncorrected. 39. Correct Choice: (A) Here the conclusionsignaled by the word thereforeis that one of the two schools in Morton will have to close. The evidence is that San Hernan will have constructed its own school. To link these, we must assume that the opening of the new school will reduce the Morton student population below what would justify keeping both schools open. So choice (A) is correct. Choices (B) and (E) do not link the opening of the new school to the attendance at the original two schools. Choices (C) and (D) actually weaken the argument, since it would be more likely that the remaining population at the original schools would still merit two schools. 40. Correct Choice: (B) Individual income went up, but household income went down. A possible explanation is that the number of individuals per household declined. Choice (B) states exactly this, and is therefore the best answer. Choices (A) and (C) both address factors that could change income levels, but neither can explain the simultaneous decline in household income and rise

42

EXPLANATIONS in personal income. Choice (D) doesnt address income at all. Choice (E) suggests that other states experienced the same phenomenon, but doesnt explain it. 41. Correct Choice: (B) The conclusion in this argument is in the first sentence: reducing employees will increase profits and improve the economy in general. The author then explains how this will happen. Choice (B) undermines this line of thinking by pointing out that what the author suggests is wrong: corporations cannot reduce staff levels and still maintain their operations. Even if the overtime hours will be less efficient than the non-overtime hours, its still possible that the employees left will be able to handle the work, so choice (A) is not correct. Choice (C) brings in irrelevant information: that the economy will substantially improve without employee reductions doesnt weaken the argument, as its possible that this authors recommendation would improve the economy more quickly and/or substantially than if no action was taken. Choice (D) simply provides a statistical result of reducing staff levels, and doesnt address the economists conclusion. Choice (E) focuses on the impact on the people who lost their jobs and got new ones. The fact that these new jobs would pay less than the previous jobs is bad for these individuals but does not address the authors conclusion.

43

Anda mungkin juga menyukai